ChaseDream
搜索
返回列表 发新帖
00:00:00

In Teruvia, the quantity of rice produced per year is currently just large enough to satisfy domestic demand. Teruvia's total rice acreage will not be expanded in the foreseeable future, nor will rice yields per acre increase appreciably. Teruvia's population, however, will be increasing significantly for years to come. Clearly, therefore, Teruvia will soon have to begin importing rice.

Which of the following is an assumption on which the argument depends?

正确答案: A

更多相关帖子

524

帖子

15

好友

4712

积分

ChaseDream

注册时间
2003-03-17
精华
8
解析
查看: 3071|回复: 5
打印 上一主题 下一主题

GWD-3-21

[复制链接]
跳转到指定楼层
楼主
发表于 2010-11-17 21:55:01 | 只看该作者 回帖奖励 |倒序浏览 |阅读模式
In Teruvia, the quantity of rice produced per year is currently just large enough to satisfy domestic demand.  Teruvia’s total rice acreage will not be expanded in the foreseeable future, nor will rice yields per acre increase appreciably.  Teruvia’s population, however, will be increasing significantly for years to come.  Clearly, therefore, Teruvia will soon have to begin importing rice.

Which of the following is an assumption on which the argument depends?

A.    No pronounced trend of decreasing per capita demand for rice is imminent in Teruvia.
B.    Not all of the acreage in Teruvia currently planted with rice is well suited to the cultivation of rice.
C.    None of the strains of rice grown in Teruvia are exceptionally high-yielding.
D.    There are no populated regions in Teruvia in which the population will not increase.
E.    There are no major crops other than rice for which domestic production and domestic demand are currently in balance in Teruvia.
C和E怎么不对了?
收藏收藏 收藏收藏
沙发
发表于 2010-11-17 22:07:18 | 只看该作者
The answer is A.

For C, irrelevant.  It's the total yields of rice that counts and the stimulus already says that the total yields of rice would not increase appreciably. If it is already stated, it is not an assumption.

For D, it just strengthens the original claim that the total population will increase.  Since we already know this trend, it is not an assumption.
板凳
 楼主| 发表于 2010-11-17 22:10:17 | 只看该作者
那么A这个假设是怎么得来的呢?
地板
发表于 2010-11-17 22:22:58 | 只看该作者
If you negate A, then the argument will fall apart since the rate of decreasing demand might offset the rate of increasing population in terms of total demand for rice.
5#
 楼主| 发表于 2010-11-17 22:31:50 | 只看该作者
If you negate A, then the argument will fall apart since the rate of decreasing demand might offset the rate of increasing population in terms of total demand for rice.
-- by 会员 sdcar2010 (2010/11/17 22:22:58)


哦,就是取非原题不成立吧,明白3Q~
6#
发表于 2010-11-17 22:56:20 | 只看该作者


For all necessary assumption questions, use 取非原题不成立 method.
您需要登录后才可以回帖 登录 | 立即注册

Mark一下! 看一下! 顶楼主! 感谢分享! 快速回复:

手机版|ChaseDream|GMT+8, 2024-9-20 01:30
京公网安备11010202008513号 京ICP证101109号 京ICP备12012021号

ChaseDream 论坛

© 2003-2023 ChaseDream.com. All Rights Reserved.

返回顶部